For polynomial lovers!

Algebra Level 5

There are nonzero integers a , b , r a,b,r and s s such that the complex number r + s i r+si is a zero of the polynomial P ( x ) = x 3 a x 2 + b x 65 P(x)=x^3-ax^2+bx-65 . For each possible combination of a a and b b , let p a , b p_{a,b} be the sum of zeros of P ( x ) P(x) . Find the sum of the p a , b p_{a,b} 's for all the possible combinations of a a and b b .

Here i = 1 i=\sqrt{-1}

This is the part of this set .


The answer is 80.

This section requires Javascript.
You are seeing this because something didn't load right. We suggest you, (a) try refreshing the page, (b) enabling javascript if it is disabled on your browser and, finally, (c) loading the non-javascript version of this page . We're sorry about the hassle.

1 solution

Ravi Dwivedi
Jul 11, 2015

Clearly the other imaginary root is r s i r-si and third root is rational and let this third root be q 'q' .

P ( x ) = ( x q ) ( x r s i ) ( x r + s i ) = x 3 a x 2 b x 65 P(x)=(x-q)(x-r-si)(x-r+si)=x^3-ax^2-bx-65 (Here i = 1 i=\sqrt{-1} )

( x q ) ( x 2 2 r x + r 2 + s 2 ) = x 3 a x 2 b x 65 \implies (x-q)(x^2-2rx+r^2+s^2)=x^3-ax^2-bx-65

Compare coefficients to get q ( r 2 + s 2 ) = 65 b = r 2 + s 2 + 2 r q a = q + 2 r q(r^2+s^2)=65 \\ b=r^2+s^2+2rq\\ a=q+2r

So each p a , b = a p_{a,b}=a so we just need to find the values of a a in each pair.

If q = 1 , r 2 + s 2 = 65 q=1,r^2+s^2=65 so ( r , s ) = ( ± 1 , ± 8 ) , ( ± 8 , ± 1 ) , ( ± 4 , ± 7 ) , ( ± 7 , ± 4 ) (r,s)=(\pm1,\pm8),(\pm8,\pm1),(\pm4,\pm7),(\pm7,\pm4)

When q = 5 , r 2 + s 2 = 13 q=5, r^2+s^2=13 so ( r , s ) = ( ± 2 , ± 3 ) , ( ± 3 , ± 2 ) (r,s)=(\pm2,\pm3),(\pm3,\pm2)

When q = 13 , r 2 + s 2 = 5 q=13,r^2+s^2=5 so in this case ( r , s ) = ( ± 2 , ± 1 ) , ( ± 1 , ± 2 ) (r,s)=(\pm2,\pm1),(\pm1,\pm2)

Only s 2 s^2 appears in the equations for a a and b b so we ignore the signs of s s .

So the sum of the p a , b = a p_{a,b}=a for q = 1 q=1 is q q times the number of distinct r r values (as each value of r r generates a pair ( a , b ) (a,b) )

Answer= ( 1 ) ( 8 ) + ( 5 ) ( 4 ) + ( 13 ) ( 4 ) = 80 (1)(8)+(5)(4)+(13)(4)=\boxed{80}

Moderator note:

Why is it clear that the third root is rational? In fact, can we immediately show that it must be an integer (under the given conditions)?

Yes it is an integer....didn't care for rational or integer when wrote down the solution

Ravi Dwivedi - 5 years, 11 months ago

Log in to reply

The point is:
1) Avoid "clearly" or "obviously" unless it's immediately obvious. If there is one-line of work that needs to be done, state how it's done.
2) After solving the problem, review through the way that you solved it, and see if there are improvements / shortening of the problem that can be done. It helps the reader if you are concise, and show the strongest possible version of the statement.

Look at Solution Writing Notes for more tips.

Calvin Lin Staff - 5 years, 11 months ago

Log in to reply

In this solution...what improvement you suggest?

Ravi Dwivedi - 5 years, 11 months ago

Log in to reply

@Ravi Dwivedi Saying "Clearly, the other imaginary root is r i s r - is . Thus, by Vieta's formula, the third root is q = a 2 r q = a - 2r . This immediately explains why q q is an integer, instead of making the reader guess why it is obvious that the third root is rational.

For example, based on what you wrote, my first thought would be "Hm, do we apply the rational root theorem to prove that there is a real rational root? How is this clear?"

Calvin Lin Staff - 5 years, 11 months ago

Answer is coming out to be 160

[2 * 15+2 * 11+2 * 17+2 * 9]+[2 * 9+2 * 1+2 * 11+2 * (-1)]+[2 * 3+2 * (-1)+2 * 17+2 * (-15)]+[2 * 3+2 * (-7)+2 * 15+2 * (-13)]
= (2 * 4 * 13) + (2 * 4 * 5) + (2 * 4 * 1)+(2 * 4 * 1) =160

Akul Agrawal - 5 years, 10 months ago

0 pending reports

×

Problem Loading...

Note Loading...

Set Loading...